A book tour will be successful if it is well publicized and the author is an established writer. Julia is an establis...

Matt on November 30, 2014

why not A

I understand C is the correct answer but why is A not correct? The only discrepancy I can notice is that A uses an 'only if' statement and the stimulus does not.

Replies
Create a free account to read and take part in forum discussions.

Already have an account? log in

Melody on December 5, 2014

Let's diagram:

"A book tour will be successful if it is well publicized and the author is an established writer."

P1: WP & EW ==> BTS
not BTS ==> not WP or not EW

"Julia is an established writer, and her book tour was successful."

P2: EW & BTS

"So her book tour must have been well publicized."

C: WP

The argument erroneously reasons by using the necessary condition "BTS" and one of the two sufficient conditions "EW" to conclude the other sufficient condition "WP." We cannot use a necessary condition to conclude anything, let alone to conclude a sufficient condition.

Let's diagram (A):

"This recipe will turn out only if one follows it exactly and uses high-quality ingredients."

P1: RTO ==> FE & HQI
not FE or not HQI ==> not RTO

"Arthur followed the recipe exactly and it turned out."

P2: FE & RTO

"Thus, Arthur must have used high-quality ingredients."

C: HQI

Unlike the first stimulus, we have two necessary conditions and only one sufficient condition. Whereas the stimulus and answer choice (C) both have two sufficient conditions and one necessary condition along with the same reasoning.

Hope that clears things up!

Matt on December 6, 2014

Great explanation- thanks!

on June 16, 2019

How did you know that they are necessary conditions? Is it the qualifier?

Ravi on June 17, 2019

@MJA7, happy you liked Naz's explanation! Let us know if you have any
other questions!

@heidiz,

We know that they're the necessary conditions because they're on the
right side of the conditional chain.

IF X, then Y

X - >Y

X is the sufficient condition, and Y is the necessary condition.

The stimulus's diagram:

WP + EW - >BTS

BTS is the necessary condition

Julia: EW and BTS

C: WP

The argument erroneously takes one sufficient condition and the
necessary condition to conclude the other sufficient condition

(C)'s diagram:

KS + Watered more than twice weekly - >they'll die

they'll die is the necessary condition

Cactus: KS and dead

C: watered more than twice weekly

Just as in the stimulus, (C) makes the same error of taking one of the
sufficient conditions and the necessary condition and concluding the
other sufficient condition. Thus, (C) is the correct answer choice.

Does this make sense? Let us know if you have any more questions!

on June 18, 2019

Thanks @ravi, was it the wording that let you know what was on the right and what was on the left?